OB/GYN Rotation

¡Supera tus tareas y exámenes ahora con Quizwiz!

A 23-year-old nulligravida comes to your office for contraception counseling. She has a seizure disorder that is well controlled on carbamazepine. She is a nonsmoker and has no other medical problems or complaints. She is currently in a relationship and does not want to get pregnant in the next several years. Which one of the following contraceptives is the most appropriate? A. Combined oral contraceptives B. Etonogestrel/ethinyl estradiol vaginal ring C. Levonorgestrel intrauterine device D. Progestin-only pills

C. Levonorgestrel intrauterine device Certain antiepileptic drugs induce hepatic metabolism of estrogen (carbamazepine, oxcarbazepine, phenobarbital, phenytoin, and topiramate). This can potentially lead to failure of any contraceptive that contains estrogen. Therefore progestin only birth control methods would be beneficial to this patient. The levonorgestrel intrauterine device and copper intrauterine device are acceptable choices even for a nulligravida.

A 29-year-old woman who is two weeks postpartum following an uncomplicated pregnancy and delivery of a full-term infant is brought in by emergency medical services with an ongoing generalized tonic-clonic seizure. Which of the following medications should be administered first? A. Labetalol B. Lorazepam C. Magnesium sulfate D. Phenobarbital

C. Magnesium sulfate A pregnant or recently postpartum patient with new-onset seizure should be considered to have eclampsia. Eclampsia refers to seizures that develop as a complication of severe preeclampsia. The clinical manifestations of preeclampsia are hypertension after 20 weeks of pregnancy plus proteinuria. Most cases of eclampsia occur in the 3rd trimester, with approximately 80% occurring during delivery or within the first 48 hours after delivery, though seizures may occur as late as several weeks postpartum. Seizures are most commonly tonic-clonic and last 60 to 90 seconds. Magnesium sulfate is the drug of choice for eclamptic seizures.

You are caring for a 33-year-old G3P2 women. Her PMH is significant for obesity and allergies. She receives routine obstetrical care. When is she due for her glucose challenge testing to rule out gestational diabetes? A. Between 12 and 16 weeks gestation B. Between 16 and 20 weeks gestation C. Between 20 and 24 weeks gestation D. Between 24 and 28 weeks gestation E. Between 28 and 32 weeks gestation

D. Between 24 and 28 weeks gestation Routine surveillance for gestational diabetes of a pregnant women is between 24 and 28 weeks; it is not modified due to obesity or other risk factors for diabetes. Gestational diabetes is a hormone-mediated intolerance. Surveillance is modified in the presence of pre-existing diabetes.

A 21-year-old female presents to clinic complaining of mild, low abdominal ache and intermittent dysuria. She denies N/V/D, and she is sexually active and uses condoms some of the time. Her LMP was 10 days ago, and she is a G0P0. Physical exam reveals a healthy female in no acute distress. Vitals are as follows: P 70, BP 120/80, T 99.9°F. Lungs are clear, CV RRR, abd soft non-tender, + BS. Pelvic exam reveals normal external genitalia, scant discharge, moderate cervical motion tenderness, and no adnexal masses. What is her most likely diagnosis? A. Tubo-ovarian abscess B. Gastroenteritis C. Ectopic pregnancy D. Cervicitis E. Pelvic inflammatory disease

D. Cervicitis Suspicion for PID should be very high in a young, healthy, and sexually active woman with cervical motion tenderness. She is not spotting and just menstruated, making ectopic pregnancy much less likely.

A 40-year-old female is status post a dilatation and curettage for hydatidiform mole. On week 3 post surgery, her follow-up quantitative hCG level has elevated slightly. What is the most likely diagnosis? A. Adenocarcinoma of the ovary B. Adenocarcinoma of the uterus C. Retained hydatidiform mole D. Choriocarcinoma E. Corpus luteum cyst

D. Choriocarcinoma 5% of hydatidiform mole progress to choriocarcinoma; the longer the mole in intrauterine the higher the risk. Pre-surgical evaluation for mole removal includes a chest x-ray to rule out distant metastasis. hCg that either plateuas or elevates is choriocarcinoma until proven otherwise, and requires prompt evaluation.

A 34-year-old woman presents complaining of dysuria and vaginal itching. Your speculum exam reveals the findings seen in the image. Which of the following statements is correct regarding this diagnosis? A. A fishy odor is present when vaginal discharge is mixed with potassium hydroxide B. Metronidazole is the recommended treatment C. Multiple petechiae are often seen on the vaginal wall D. The pH of the discharge is less than 4.5 E. Vaginal discharge is often foul smelling

D. The pH of the discharge is less than 4.5 Candidal vaginitis is characterized by a thick, curdy white (cottage cheese-like) discharge. Patients typically complain of vaginal itching, dysuria, and dyspareunia. Risk factors include diabetes, HIV, recent antibiotic use, and pregnancy. A microscopic slide prepared with 10% potassium hydroxide (KOH) will reveal characteristic branch chain hyphae (pseudohyphae) and spores. The pH of the discharge is less than 4.5, whereas the pH of the other causes of vaginitis is greater than 4.5.

A 25-year-old woman comes in for her annual physical examination and renewal of her oral contraceptive prescription. She has no problems today and appears fit and well. On palpation of the thyroid, the right lobe is small, smooth, and free of nodules, but appears to be slightly larger than the left lobe. She has no other unexpected physical findings. The enlargement most likely represents which of the following? A. inflammation of the thyroid B. goiter C. hypothyroidism D. malignancy E. normal finding

E. normal finding Slight enlargement of one lobe of the thyroid gland in a healthy woman with no complaints is most likely a normal finding. In patients with an inflammation of the gland (A), i.e., thyroiditis, and hypothyroidism (C) the thyroid is usually diffusely enlarged and may be markedly asymmetric. Other findings depend upon the cause. Goiters (B) are typically nodular and may be quite enlarged. Thyroid malignancy (D) usually presents as a firm, non-tender nodule.

A 30-year-old G1P0 woman who is 15 weeks pregnant undergoes "triple screening." The maternal alphafetoprotein (AFP), human chorionic gonadotropin (hCG), and unconjugated estriol (uE3) levels are all lower than normal. This suggests which of the following possible problems with the fetus? A. cystic fibrosis B. Down syndrome C. homocystinuria D. G6PD deficiency E. trisomy 18

E. trisomy 18 The triple screen detects possible Down syndrome and trisomy 18. When the fetus has Down syndrome (B), the AFP and uE3 are low and the hCG is high, while trisomy 18 is suggested by low values in all three. A positive screen must be followed up by fetal karyotyping. Cystic fibrosis (A), homocystinuria (C), and G6PD deficiency (D) are not detected by the triple screen.

A 27-year-old female is 8 weeks postpartum with her first child and has been exclusively nursing since discharge at the hospital. She has a 5-day history of engorgement in her right breast, which is red, tender, and feels warm to the touch. She states she is feverish but has not taken her temperature. On physical examination you see the breast as shown below. What is the usual causative agent of your suspected diagnosis? A. Staphylococcus aureus B. Streptococcus pyogenes C. Group A Beta-hemolytic streptococcus D. Fungal E. Group B streptococci

A. Staphylococcus aureus Staphylococcus aureus is usually the causing agent in puerperal mastitis. Streptococcus infection can cause mastitis but is much less frequent than staph infections. Streptococcus pyogenes (B) is the cause of Group A beta-hemolytic streptococcus (C), which may cause necrotizing fasciitis, among other infections. Group B streptococci (E) is commonly seen colonized in the lower genital tract of females. It is a common cause of neonatal sepsis. Fungal infections (D) of the breast would not typically present in this way.

You are taking care of a 32-year-old G2P1 at 39 weeks gestation in active labor. Her pregnancy is complicated by gestation diabetes. The fetal head delivered, but the anterior shoulder did not deliver with gentle downward traction. What would be the next most appropriate action? A. More forceful traction and fundal pressure B. Call for assistance and McRoberts maneuver C. Call for assistance and more forceful traction D. Call for help and fundal pressure

B. Call for assistance and McRoberts maneuver Shoulder dystocia is an obstetrical emergency, and help should always be summoned. The McRoberts maneuver increased the AP diameter, thus accommodating a large head; subrapubic pressure can help dislodge the anterior shoulder, but simple fundal pressure continues to impact it against the pelvic bone.

A 30-year-old woman presents with fever and abdominal pain. She is 3 days postpartum after cesarean section. Physical examination reveals lower abdominal tenderness to palpation and foul smelling vaginal discharge. What management is indicated? A. Ceftriaxone IM and Azithromycin PO B. Clindamycin IV + gentamycin IV C. Fluconazole D. Metronidazole

B. Clindamycin IV + gentamycin IV This patient presents with endometritis and should be treated with broad-spectrum antibiotics and admitted to the hospital. Endometritis affects 1 in 20 vaginal deliveries and 1 in 10 cesarean sections. There are a number of associated risk factors including operative delivery, prolonged rupture of membranes, lack of prenatal care and frequent vaginal examinations. Endometritis is a polymicrobial infection with gram-positive cocci and gram-negative coliforms involved. Patients typically present with abdominal pain, fever and foul-smelling lochia or discharge. It commonly develops the second or third day post partum. Clindamycin IV and gentamycin IV are typically recommended.

A 24-year-old female, with a history of type 2 diabetes, presents with the inability to conceive after 14 months of unprotected sexual intercourse with her husband. Her vital signs are unremarkable and you calculate a BMI of 31. Physical examination reveals acne vulgaris and hirsutism. Which of the following treatment options for her infertility would be the most effective considering your suspected diagnosis? A. Medroxyprogesterone acetate B. Clomiphene citrate C. Metformin D. Spironolactone E. Mini-pill (progestin only)

B. Clomiphene citrate Clomiphene citrate is highly effective as the first line treatment for infertility in PCOS. It can be accompanied with metformin, weight loss, exercise, and exogenous gonadotropins when clomiphene fails. PCOS in over half of patients is accompanied with obesity, abnormalities in insulin control, metabolic syndrome, and infertility. Medroxyprogesterone acetate (A) and the mini-pill (E) are used for endometrial protection and with oral contraceptive pills. Metformin (C) will help with her diabetes. Spironolactone (D) is a diuretic, which acts as a weak androgen receptor antagonist.

A 55-year-old postmenopausal woman presents to your office with a complaint of vaginal bleeding. Which of the following is the most appropriate next step in management? A. Abdominal ultrasound B. Endometrial biopsy C. Hysterectomy D. Watchful waiting

B. Endometrial biopsy After a careful history and physical exam, initial diagnostic testing to rule out endometrial cancer involves either endometrial biopsy or transvaginal ultrasound. Advantages of an endometrial biopsy include its high sensitivity, low cost and low incidence of complications.

A 23-year-old woman presents to clinic complaining of amenorrhea for 3 months. She also complains of increasing facial hair and weight gain. On exam, PMH: menarche age 13. Physical exam reveals a well-developed, slightly obese female with a BMI of 29. Her amenorrhea can likely be improved with which therapy? A. Testosterone administration B. Progesterone administration C. Estrogen administration D. Estrogen suppression E. Testosterone suppression

B. Progesterone administration Progesterone administration slows GnRH pulses, thereby improving FSH secretion and follicular maturation.

A 14-year-old otherwise healthy boy presents to your clinic with his father. He is concerned because his breasts are enlarged. Appropriate management includes which of the following? A. BRCA gene testing B. Reassurance C. Serum estrogen level D. Ultrasound

B. Reassurance The patient has gynecomastia, a common finding in males during early pubertal changes (occurs in half of men at some point during adolescence, most commonly between ages 13-14 years or tanner stage 3-4). Gynecomastia results from relative imbalance between estrogen and testosterone levels. Gynecomastia in males and asymmetric breasts in females are a normal aspect of pubertal development and is a normal finding. Therefore, reassurance is all that is required. Further evaluation of the breast tissue for causes of pathologic gynecomastia is indicated when gynecomastia has persisted beyond 2 years or over 17 years of age.

A 17-year-old girl is seen in clinic due to complaints of excessive body hair. She denies taking any medication. She has irregular menses and denies sexual activity. On exam, her BMI is 31, with moderate hirsutism on upper lip and chest, moderate acne on her face, Tanner 5 breasts and pubic hair. The rest of her exam findings are normal. Which of the following is an expected laboratory finding? A. Increased cortisol B. Increased follicle stimulating hormone C. Increased luteinizing hormone D. Increased thyroid stimulating hormone

C. Increased luteinizing hormone The patient has manifestations of hyperandrogenism and menstrual abnormalities that are suspicious for polycystic ovary syndrome (PCOS). PCOS is characterized by the triad of oligo-ovulation or anovulation, clinical or biochemical hyperandrogenism, and ovarian cysts (greater than or equal to 12 follicles). There is gonadotrophic dysregulation with increased luteinizing hormone (LH) pulsatility and abnormally high ratios of circulating LH to follicle stimulating hormone (FSH).

A 27-year-old woman and her male partner come to the emergency department for assistance with emergency contraception. They experienced condom failure during intercourse an hour ago and neither desires pregnancy. Her last menstrual period was approximately two weeks ago and her cycles occur every 28-30 days. Her medical history includes a deep venous thrombosis during labor and delivery 5 years ago. What is the most appropriate course of action at this time? A. Administration of an ethinyl estradiol and levonorgestrel combination now and in 12 hours B. Dilation and curettage C. Insertion of a copper-containing intrauterine device D. Serial beta hCG determinations E. Testing for factor V Leiden

C. Insertion of a copper-containing intrauterine device Insertion of a copper-containing IUD is an effective means of preventing an unintended pregnancy in this case.

A 25-year-old gravida 1 woman who is HIV positive arrives at the hospital in early labor. Membranes are intact and the cervix is 50% effaced and 3 to 4 cm dilated. Fetal heart rate is 150 beats/min. Which of the following procedures is contraindicated during labor? A. amniotomy B. augmentation of labor with oxytocin C. external monitoring D. operative delivery E. use of fetal scalp electrodes

E. use of fetal scalp electrodes Use of fetal scalp electrodes and scalp sampling is contraindicated in the HIV-positive woman because it increases the risk of vertical transmission of the human immunodeficiency virus to the infant. While ruptured membranes for more than 4 hours is associated with an increased risk of vertical transmission, amniotomy per se is not contraindicated. Augmentation of labor, external monitoring, and operative delivery are not contraindicated and, in fact, may be indicated in specific instances for the well-being of the infant and/or mother.

Normal Pregnancy Physiology

↑: Blood volume, CO, TV ↓: FRC, SVR Respiratory alkalosis: 3rd trimester Doppler heart tone at 10 weeks HR: ↑ 10-15 bpm BP: ↓ in 2nd trimester, normalizes in 3rd Serum ß-hCG: doubles every two days in early pregnancy Fundal height: 12 weeks: pubic symphysis 20 weeks: umbilicus 20-32 weeks: height (cm) above symphysis = gestational age (weeks) Transvaginal ultrasound: IUP visualized when ß-hCG > 1500 Transabdominal ultrasound: IUP visualized when ß-hCG > 4000

A 25-year-old sexually active woman presents to the Emergency Department with a complaint of a painful vulvar ulcers and a swollen inguinal lymph node. She denies dysuria and vaginal discharge. Which of the following is most consistent with the description of the offending infectious organism? A .Gram negative bacillus B. Gram negative diplococcus C. Obligate intracellular gram negative organism D. Spirochete

A .Gram negative bacillus Chancroid is a common sexually transmitted disease in developing nations, but is relatively rare in the United States. Chancroid is caused by Haemophilus ducreyi, a gram negative bacillus, which has an incubation period of 4 to 10 days. Initially, a small pustule will form at the site of inoculation and progresses to multiple, painful ulcerations with sharply demarcated purulent bases. The number of ulcers and the pain associated with the ulcers are the distinguishing features of chancroid and differentiate it from syphilis. Chancroid is also associated with painful inguinal lymphadenopathy, termed a bubo, which is unilateral, large, painful, fluctuant and may become suppurative. Treatment is with Azithromycin 1 gm orally once, ceftriaxone 250 mg intramuscularly once, ciprofloxacin 500 mg orally twice a day for three days, or erythromycin 500 mg orally four times a day for 7 days.

A 17-year-old G1P0 woman at 25-weeks gestation presents with intermittent blurred vision. On presentation, she is asymptomatic. Vital signs are HR 84, BP 175/113, oxygen saturation 97%. Physical examination reveals 2+ pitting edema on both lower extremities and urinalysis has 3+ protein on dip. Which of the following is most likely indicated? A. Admit for further obstetrics evaluation B. Antiepileptic medications C. Arrange follow up with the patient's obstetrician D. Emergency cesarean section

A. Admit for further obstetrics evaluation This patient presents with severe preeclampsia and should be admitted for further obstetric evaluation. Preeclampsia is defined as gestational hypertension (>140/90 mm Hg) with proteinuria (>300 mg/24 hr) that occurs after 20-weeks gestation. Progression from preeclampsia to eclampsia (hypertension, proteinuria and seizures) is unpredictable and can occur rapidly. In preeclampsia, patients may be asymptomatic. In severe disease, typically defined as a blood pressure >160/110 mm Hg, patients may have associated epigastric or liver tenderness, visual disturbances or severe headaches. These patients should be admitted for further management. Treatment for these patients is the same as in eclampsia. The goal of treatment is prevention of seizures or permanent maternal organ damage. Magnesium should be given for seizure prophylaxis.

The presence of which of the following best differentiates premenstrual syndrome from premenstrual dysphoric disorder? A. Anger and irritability B. Confusion and other cognitive changes C. Mood swings and other mood changes D. Serotonin dysfunction

A. Anger and irritability Premenstrual syndrome (PMS) is not clearly defined, as its high prevalence, unclear etiology and myriad of symptoms makes it difficult to classify it as a disease or a cluster of physiologic changes. Age of presentation is mainly in the late 20s to early 30s, and it recurs in up to 75% of affected women. Etiological theories include psychological disturbance, alterations in estrogen and progesterone balance as well as serotonin function, hypoglycemia and hyperprolactinemia. Symptoms include headache, insomnia, fatigue, low energy, bloating, breast tenderness, abdominopelvic pain, depression, anxiety, dysphoria, mood lability, appetite changes, crying episodes, confusion, poor coordination and poor concentration. Symptoms typically interfere with the woman's daily life. According to the American Psychiatric Association DSM-IV, prominent anger, irritability and internal tension associated with severe premenstrual syndrome symptoms is defined as premenstrual dysphoric disorder

A 15-year-old woman presents to the office with her mother concerned that she has not had a menstrual cycle. She is an avid runner, logging 20 miles per week. On exam she exhibits no breast development or axillary or genital hair. Her mother was 15 when she started her menstrual cycle. What is your next step? A. Begin work-up for primary amenorrhea B. Encourage her to stop exercising for 3 months C. Induce cycle with medroxyprogesterone (Provera®) D. No work up, but provide education on the topic

A. Begin work-up for primary amenorrhea Primary amenorrhea is defined as failure of menarche by age 16 in a woman with apparently normal sexual development or by age 14 in a woman with no secondary sexual characteristics. Secondary amenorrhea is failure of menstruation after normal menses are established, with the caveat that at least 3 months have passed with apparently normal menses or 9 months have passed in a woman with oligomenorrhea. Primary amenorrhea can be caused by obstruction of the outflow tract, androgen insensitivity, gonadal dysgenesis, hyperprolactinemia, and dysfunction of the hypothalamus, pituitary, or thyroid. To evaluate a patient with primary amenorrhea, after a thorough clinical history, the physical examination must focus on development of secondary sexual characteristics (breast development, pubic, and axillary hair).

Which of the following tests should an HIV-positive pregnant woman undergo in each trimester of pregnancy? A. CD4+ lymphocyte count B. cytomegalovirus serology C. postpartum depression with controls D. shielded chest radiography E. venereal disease research laboratory (VDRL)

A. CD4+ lymphocyte count HIV-positive pregnant women should undergo CD4+ serology each trimester. Early in the pregnancy, they should undergo shielded chest radiography, CMV baseline testing, and tuberculosis testing with controls. Syphilis testing should be completed initially and as usually recommended later in pregnancy.

A woman presents with fever and foul-smelling vaginal discharge 3 days after delivery of a full-term fetus. She is febrile, with uterine tenderness on pelvic exam. Which of the following is the strongest risk factor for postpartum endometritis? A. Cesarean section B. Internal fetal monitoring C. Multiple gestation D. Premature rupture of membranes

A. Cesarean section Postpartum endometritis is the most common puerperal infection, usually developing on the 2nd or 3rd day postpartum. Typically, the lochia has a foul odor, and the patient develops a leukocytosis. The infection begins in the endometrium and can extend to the myometrium or parametrium. It is a serious infection that can lead to complications such as peritonitis, septic thrombophlebitis, and necrotizing fasciitis. The pathogens involved are typically the flora of the bowel, perineum, vagina, and cervix. The strongest risk factor for endometritis is cesarean section.

A 21-year-old woman with no prenatal care presents for evaluation of lower abdominal pain and fever. She estimates that she is approximately 7.5 months pregnant. On questioning, she acknowledges intermittent pain for two days and a gush of fluid shortly after the pain began. Her temperature is 101.8°F. Physical examination is notable for purulent material in the vaginal vault. Which of the following is the most likely diagnosis? A. Chorioamnionitis B. Endometritis C. Pelvic inflammatory disease D. Urinary tract infection

A. Chorioamnionitis Beginning at 16 weeks, the membranes of the chorioamniotic sac adhere to the cervical os and are at risk for infection. Chorioamnionitis is an intra-amniotic infection of the chorion and amniotic layers of the amniotic sac. The placenta and fetal membranes may also be involved. It is caused by an ascending infection of normal vaginal flora. This is a clinical diagnosis and patients require intravenous antibiotics, most commonly ampicillin and gentamicin.

A 54 year-old female returns for pathology results after being diagnosed with ovarian cancer. Which of the following is the most likely cause of ovarian epithelial malignancy? A. Serous tumors B. Endometrioid tumors C. Mucinous tumors D. Clear cell tumors E. Transitional cell tumors

A. Serous tumors The most common of the ovarian epithelial malignancies are serous tumors (50%); tumors of mucinous (25%) (C), endometrioid (15%) (B), clear cell (5%) (D), and transitional cell (E) histology or Brenner tumors (1%) represent smaller proportions of epithelial ovarian tumors. Over half of all epithelial ovarian cancers have serous histology. The second most common histologic type of epithelial ovarian cancers are endometrioid adenocarcinomas.

A 26-year-old previously healthy woman presents to the Emergency Department with abdominal pain. She was at home when she developed sudden onset lower abdominal pain followed by a brief syncopal episode. Her vital signs include blood pressure of 88/46 mm Hg, heart rate of 112 beats/minute, respiratory rate of 18 breaths/minute, temperature of 37.6°C, and oxygen saturation of 98%. She had a positive home pregnancy test yesterday. After initiating aggressive resuscitation, what is the most appropriate next step in management? A. Consult Obstetrics and Gynecology B. Obtain a complete blood count C. Obtain a serum human chorionic gonadotropin (hCG) level D. Pelvic ultrasound

A. Consult Obstetrics and Gynecology A positive pregnancy test with an unknown location of implantation does not confirm the diagnosis, but whenever an intrauterine pregnancy cannot be confirmed, ectopic implantation should be considered. Due to the life-threatening nature of the illness, any hypotensive patient with strong suspicion for an ectopic pregnancy warrants an emergent consult to Obstetrics and Gynecology for possible operative management. Although obtaining a complete blood count (B), serum human chorionic gonadotropin levels (C), and a pelvic ultrasound (D) can aid in the diagnosis of ectopic pregnancy, in an unstable patient with high suspicion for ectopic pregnancy these diagnostic test should not delay mobilization of resources that can provide definitive care.

A 37-year-old woman, G3P2 at 30 weeks gestation, complains of lower extremity swelling and her weight is up 5 pounds this week. Her PMH is insignificant, and her other pregnancy was uncomplicated. Her BP baseline is now 142/92. On exam her BP is unchanged, her UA shows 2+ protein, and FHTs are 152. What is the cause of the protein in her urine? A. Glomeruloendotheliosis B. Glomerulonephritis C. Renal vasospasm D. Glomerular hemorrhage E. Glomerular infarct

A. Glomeruloendotheliosis The classic histological change that occurs in the renal system (in preeclampsia) is swelling and inflammation of the endothelium and of the glomeruli, which leads to endothelial leaking.

A 25-year-old woman presents to your office with amenorrhea, abdominal cramping, and irregular vaginal bleeding. She usually has monthly menstrual cycles, but has been intermittently spotting over the last 2 months. Her exam shows a firm uterus without tenderness and a dilated cervix with blood in the vaginal vault. Her beta HCG is positive and a bedside ultrasound reveals a yolk sac, fetal pole and no evidence of cardiac activity. What is the most likely diagnosis? A. Inevitable abortion B. Physiologic bleeding C. Septic abortion D. Threatened abortion

A. Inevitable abortion Inevitable abortion is associated with painful abdominal cramps, vaginal bleeding and a dilated cervix with gestational tissue often visible at the cervical os or on ultrasound. Dilatation and curettage is indicated for significant cramping or blood loss. First-trimester miscarriage is a common event. All women with clinically recognized pregnancies have approximately a 15% chance of spontaneous pregnancy loss in the first 3 months of pregnancy.

Which of the following is the most common type of invasive breast cancer in women? A. Infiltrating ductal B. Infiltrating lobular C. Medullary D. Papillary

A. Infiltrating ductal Breast cancer is the leading cause of cancer death in women worldwide and is the most common type of cancer among women in the United States. Screening of women ages 50-75 is recommended every two years with screening mammography. The two types of in situ carcinomas are ductal and lobular. Infiltrating carcinomas have several different histological subtypes with infiltrating ductal carcinoma being the most common.

A 19-year-old G1 P0 presents to the emergency department complaining of abdominal pain. Her LMP was 2 months ago, and she has been spotting for the last two days. Her HCG is positive, and transvaginal ultrasound reveals no intrauterine gestational sac. The patient has an ectopic pregnancy, which has likely implanted in the fallopian tube. Why is the fallopian tube the most common non-uterine implantation site? A. Lack of sub-mucosal layer in the fallopian tube B. Inflammatory response in the endometrium C. Proliferative endometrium D. Extra myometrial tissue (fibroids) E. Excessive trophoblastic proliferation

A. Lack of sub-mucosal layer in the fallopian tube The fallopian tube is the most common site of ectopic pregnancy, accounting for over 95% of ectopic pregnancies. The lack of a submucosal layer allows for easy wall access and implantation of the fertilized ovum. The increasing rate of Chlamydia infections in the U.S. also impacts the physical anatomy of the fallopian tube, and impacts where the ovum implants. Excessive trophoblastic activity does not impact where the ovum implants, and proliferative myometrium enhances uterine implantation if the fertilized ovum makes it to the uterine cavity.

Septic arthritis in adults younger than 30 years is usually caused by A. Neisseria gonorrhea B. Staphylococcus aureus C. Pseudomonas aeruginosa D. Streptococcus pyogenes E. Salmonella species

A. Neisseria gonorrhea In patients younger than 30 years, gonococcus is the most common cause of septic arthritis. When all patients are considered, Staphylococcus aureus is the most common cause. Patients with prevalent joint disease and intravenous drug users are especially susceptible to Staphylococcus. Pseudomonas is also a common cause of septic arthritis in intravenous drug users. Salmonella is not a common cause of joint infection.

A 33-year-old G1P0 presents for evaluation of her inability to conceive a pregnancy for six months. She menstruates monthly. Her past medical history is significant for PID x 2, for which she was hospitalized for IV antibiotics. Her Chlamydia, GC, and pap smear are normal. When should a more comprehensive evaluation for her infertility begin? A. Now B. 6 months C. 12 months D. 18 months E. 24 months

A. Now A comprehensive work up should begin now, due to her advancing age and history of significant PID, which may require surgical treatment.

A patient seen at the prenatal clinic develops Graves disease at 25 weeks' gestation. Which of the following is the most appropriate treatment? A. PTU 100 mg po tid B. methimazole 10 to 30 mg po qd C. propranolol 80 mg po qid D. radioactive iodine therapy (RAI, 131I) E. levothyroxine 0.1 mg po qd

A. PTU 100 mg po tid In nonpregnant patients, PTU and methimazole are the drugs of choice for the management of Graves disease. During pregnancy, PTU has a lower incidence of crossing the placental barrier than does methimazole. It also is excreted into breast milk to a lesser degree than is methimazole. Propranolol will help with the symptoms of Graves but not treat it. It can also cause low birth weight in the infant. RAI is contraindicated in pregnancy. Levothyroxine will worsen a Graves patient's hyperthyroidism.

A 17-year-old female presents to your office with intermittent menstrual pain. She is sexually active with a single male partner, consistently utilizing condoms. She reports that she has had this pain before, most commonly two weeks before her period, and that it has been increasing in severity over the past few months. You perform a pelvic examination and she has no lesions, discharge, or discomfort on bimanual and speculum exam. She is urinary chorionic gonadotropin (UCG) negative. Of the following, what is the most appropriate treatment for this adolescent? A. Tell her she has mittelschmerz and prescribe an anti-inflammatory p.r.n. B. Tell her she has primary dysmenorrhea and send her home on pain medications. C. Recommend that she see a gynecologist to have an exploratory laparoscopic surgery to rule out endometriosis. D. Refer her for a vaginal ultrasound to rule out uterine fibroids. E. Draw a serum β-hCG.

A. Tell her she has mittelschmerz and prescribe an anti-inflammatory p.r.n. Midcycle pain (mittelschmerz) is common in women with regular menstrual periods who are not taking birth control pills. These patients may commonly have midcycle spotting caused by an estrogen surge. There is no fever and no other abnormal bleeding such as that resulting from trauma to the cervix (e.g., coitus, douching). Pain usually occurs over several cycles. There is no history of intermittent lower abdominal pain. Examination at the time of mittelschmerz may reveal some lower quadrant tenderness with or without rebound. Bimanual examination may show localized tenderness. A palpable ovary may be present, but a history of regular menses, lack of fever, and negative pregnancy tests confirm the diagnosis. Mild analgesics, especially nonsteroidal anti-inflammatory drugs (NSAIDs), and reassurance are usually adequate for these patients.

A 45-year-old woman presents with weight gain, fatigue, dry skin, constipation, and oligomenorrhea. On physical exam, bradycardia and slow deep tendon reflexes are noted. Her free T 4 is low and TSH is elevated. Which of the following medications may be responsible for her condition? A. amiodarone B. beta-blockers C. levadopa D. hydrocholorthiazide

A. amiodarone Hypothyroidism is reported in up to 10% of patients taking amiodarone, an antiarrhythmic medication. With the high iodine content of the medication and the structural similarities to thyroxine, thyroid abnormalities occur. Common side effects of amiodarone include bradycardia and constipation, so laboratory evaluation for thyroid dysfunction must be used.

A sexually active 19-year-old woman presents with clusters of painful vesicles on an erythematous base on the vulva and cervix, accompanied by temperature of 100°F and mild malaise. She reports a history of a similar outbreak last month, which resolved in 10 days. Microscopic examination of cells from the basement of a blister treated with Giemsa stain is likely to reveal A. multinucleated giant cells B. gram-positive cocci in clusters C. gram-positive cocci in chains D. gram-negative rods E. hyphae and buds

A. multinucleated giant cells The clinical presentation is consistent with herpes simplex. The appropriate microscopic study is a Tzanck smear, prepared by staining cells from the floor of a vesicle using Papanicolau, Giemsa, or Wright methods. The Tzanck smear will show multinucleated giant cells. It has a sensitivity of 60% to 70% and as a result should be confirmed by viral culture. Gram-positive cocci are consistent with staphylococcal or streptococcal infection and gram-negative rods are usually enteric pathogens. Hyphae and buds are seen on KOH prep with candidal infection.

A 23-year-old woman presents with complaints of pelvic discomfort and a vaginal discharge for the past 3 days. She finished her period last week. She is taking oral contraceptives as directed. Her medical history is significant for a therapeutic abortion with no other hospitalizations or pregnancies. She has had three sexual partners in the past 6 weeks and does not use condoms. Her most recent partner reported that he was treated recently for gonorrhea. On examination, she has a mucopurulent discharge with "strawberry" cervix on speculum examination. After collecting the appropriate specimens, the best therapeutic option for this patient is A. ofloxacin 400 mg i po × 1 dose plus azithromycin 250 mg iiii po × 1 dose B. fluconazole 150 mg i po × 1 dose C. metronidazole 500 mg iiii po × 1 dose D. ceftriaxone 250 mg IM × 1 dose

A. ofloxacin 400 mg i po × 1 dose plus azithromycin 250 mg iiii po × 1 dose Clinical presentation is consistent with cervicitis in a young woman with risk factors for sexually transmitted infection. She has likely been exposed specifically to Neisseria gonorrhea. Coinfection with Chlamydia trachomatis is common. While test results are pending, the Centers for Disease Control and Prevention in its 2006 Guidelines for STD treatment recommend treating for both with single doses (improved compliance) of ofloxacin and azithromycin first-line. In areas with quinolone resistance, intramuscular ceftriaxone is an option but coverage for Chlamydia is still necessary. Metronidazole is the appropriate therapy for trichomoniasis and fluconazole for vaginal candidiasis.

A 22-year-old sexually active woman presents for her annual gynecologic evaluation. She reports one partner for the past 6 months and takes oral contraceptive pills as directed. Her periods have been regular. Her examination is unremarkable and her Pap smear returns with atypical squamous cells of undetermined significance and positive for human papillomavirus-16. The next most appropriate step for this patient is to A. proceed with colposcopy B. repeat Pap smear in 12 months C. repeat Pap smear in 24 months D. schedule her for a loop electrosurgical excision procedure (LEEP)

A. proceed with colposcopy Human papillomavirus subtypes 6, 11, 16, and 18 increase risk for the development of cervical cancer. In a young woman over 21 years old with atypical squamous cells of undetermined significance and positive HPV 16 subtype, the next step in evaluation is the colposcopic evaluation. Alternatively, she could be followed with Pap smears at 6 and 12 months. The LEEP procedure is indicated for those with recurrent histologic finding of cervical intraepithelial neoplasm grade 2 or 3.

A woman who is pregnant suffered a spontaneous abortion at 12 weeks gestation. She is now a G2P1Ab1 and is Rh negative. When should she receive her next Rhogam (Rho D immune globulin) shot? A.Now B. In one month C. At conception of her next pregnancy D. 28 weeks gestation of next pregnancy E. After delivery of her next viable infant

A.Now Placental implantation occurred and separated with the spontaneous miscarriage. Therefore, there is a slight chance of isoimmunization, so Rhogam should be given now so that the mother does not develop antigens that can cross the placenta during the first half of the next pregnancy.

A 29-year-old G3P2 presents to the office for her obstetrical visit. She is currently 16 weeks gestation by ultrasound. Her pregnancy has had no complications to date. She is a non-smoker and takes her prenatal vitamin. What routine obstetric lab should be offered at this visit? A. Amniocentesis B. Alpha fetal protein screen C. 3D ultrasound D. D. Glucose challenge E. E. HIV testing

B. Alpha fetal protein screen AFP testing is only available between 15 and 20 weeks gestation. HIV is done on initial visit and amniocentesis is offered for risk factors, advanced age, or abnormal AFP. 3D ultrasound is not routine standard of care, and diabetic screening is done between 24 and 32 weeks gestation.

A 28-year-old G1P0 at 37.5 weeks gestation complains of a thin, watery discharge for the last 5 hours. She has soaked 3 pads. She has no pain and the fetus continues to be active. An external fetal monitor reveals heart tones in the 140s, with variability and no contractions. The pH of the vaginal fluids is 8. What does this indicate? A. Normal vaginal secretions B. Amniotic fluid C. Yeast vaginitis D. Bacterial vaginitis E. Urine

B. Amniotic fluid Premature rupture of membranes is the rupture of membranes before the onset of labor (within 2 hours); preterm rupture is the rupture prior to 37 weeks gestation. The absence of contraction on the monitor, in addition to no complaints indicates no labor. Urinary incontinence is common, but the ph of 8 indicates amniotic fluid.

A 37-year-old female presents to the labor and delivery department complaining of intermittent pain and contractions. Upon arrival, she also complains of vaginal bleeding. She is a G3P2 at 39 weeks gestation; no other prenatal complications are noted. She is a non-smoker. A physical exam reveals the following: P 90, BP 130/80, T 98.7°F, abdomen gravid, positive bowel sounds, and left lower quadrant tenderness noted. A sterile speculum exam reveals the cervix to be dilated 8, fetus is cephalic, and membranes are intact. The fetal monitor reveals heart tones in the 140s with mild, decreased variability and good quality contractions noted. Delivery is felt to be imminent, and vaginal delivery has been determined to be the best course of action. What will likely decrease bleeding and shorten time to delivery? A. Increased activity level B. Amniotomy C. Oxytocin therapy D. Epidural placement E. IV sedation

B. Amniotomy If the fetus is mature and vaginal delivery (versus c-section) has been determined to be the best course of action, then amniotomy may diminished amnionic fluid volume. This might also allow for better spiral artery compression, and serve to both decrease bleeding from the implantation site and reduce entry of thromboplastin into the maternal circulation.

A 15-year-old girl presents to clinic with vaginal discharge. She recently became sexually active but states that her partner does not have any symptoms of infection. Speculum examination reveals white discharge and an erythematous cervix. Bimanual examination is unremarkable. Urine PCR is positive for Chlamydia trachomatis. Which of the following is the most appropriate treatment options? A. Azithromycin B. Azithromycin and ceftriaxone C. Ceftriaxone, amoxicillin and metronidazole D. Ciprofloxacin and ceftriaxone

B. Azithromycin and ceftriaxone The most common manifestation of chlamydial infection in females is cervicitis and in males, urethritis. Infection may be asymptomatic or may result in vaginal or urethral discharge. The treatment of choice for C.trachomatis cervicitis is a single dose of one gram of azithromycin. In non-pregnant women, seven days of doxycycline is an alternative therapy.

A 32-year-old woman presents with fever and lower abdominal pain. She has a history of pelvic inflammatory disease. Her vitals are T 38.4°C, HR 133, and BP 101/60. On examination, the patient is toxic appearing and has marked lower abdominal tenderness to palpation with rebound and guarding. Pelvic examination reveals cervical motion tenderness, scant discharge, and left adnexal tenderness. The patient's urine beta-hCG is negative. A transvaginal ultrasound is performed and reveals a complex cystic, thick-walled, well-defined mass in the left adnexa. Which of the following is the most appropriate next step in management? A. Administer ceftriaxone and discharge home with a 14-day course of doxycycline B. Begin intravenous antibiotics and admit for possible drainage C. Obtain a CT scan to rule out appendicitis D. Send a serum beta-hCG to rule out ectopic pregnancy

B. Begin intravenous antibiotics and admit for possible drainage A tubo-ovarian abscess (TOA) typically results as a complication of pelvic inflammatory disease (PID) and is most commonly seen in sexually active women. Since it is a complication of PID, patients typically present with lower abdominal and pelvic pain, fever, vaginal discharge, and cervical motion or adnexal tenderness. Pelvic exam may reveal a palpable mass in the adnexa. Ultrasound is the test of choice for suspected TOA, and transvaginal is best for visualizing the adnexa. Treatment involves administration of intravenous antibiotics. Some abscesses require surgical drainage. Tubo-ovarian abscesses can result in irreversible tubal and ovarian damage and pose a serious threat to fertility.

A 49-year-old woman has had irregular menses for more than two years. Most of the time she requires only 1-2 mini-pads per day to handle the flow, but recently she soaked through a dozen maxi-pads in a day. On physical examination, she appears well and her pelvic exam in unremarkable. Her serum progesterone is low and her hematocrit is 39%. A transvaginal ultrasound reveals an endometrial stripe of 11 mm. Which of the following is the most appropriate next step in her management? A. Abdominal hysterectomy B. Endometrial biopsy C. Fractional curettage D. Hysteroscopy E. Watchful waiting

B. Endometrial biopsy This woman's excessive bleeding and endometrial stripe > 5 mm raises the suspicion for endometrial hyperplasia or cancer. The next step is an endometrial biopsy, which can be performed without anesthesia in the outpatient setting. Hysterectomy (A) is inappropriate without first establishing a diagnosis for the bleeding. Fractional curettage (C) is the definitive procedure for diagnosis of endometrial carcinoma, but requires anesthesia with its attendant risks. Hysteroscopy (D) is more invasive than endometrial biopsy and may spread tumor cells into the peritoneal cavity. Watchful waiting (E) is inappropriate given with size of the endometrial stripe.

A 51-year-old female presents to the office complaining of intermittent vaginal spotting for three months. She has a history of well-controlled hypertension. She is a nonsmoker. Her LMP was two years ago. Her family history is significant for colon cancer. Physical exam and pelvic exam were performed and unremarkable. What is the best diagnostic step in evaluating her vaginal bleeding? A. Transvaginal ultrasound B. Endometrial sampling C. Pap smear D. Pregnancy test E. STD testing

B. Endometrial sampling In postmenopausal women with a family history of colon cancer, there is a 30% risk of endometrial cancer. Pregnancy and STD are less likely, and do not exclude endometrial cancer. Pap smear and ultrasound are useful but can be negative. Endometrial sampling is required to confirm or rule out cancer.

Which of the following best describes an inevitable abortion? A. First trimester bleeding and a closed internal cervical os B. First trimester bleeding with an open internal os C. Parts of the product of conception have been passed and may be visible in the cervical os or the vaginal canal D. Retention of a nonviable intrauterine pregnancy within the uterus, no cardiac activity, and a closed cervical os

B. First trimester bleeding with an open internal os Approximately 80% of miscarriages occur during the first trimester; the rest occur before 20 weeks of gestation or when the fetus is <500 g, considered premature birth. Approximately 25% of pregnant patients experience bleeding. Approximately 50% of all women who have bleeding during early pregnancy miscarry. Those with a history of bleeding who do not miscarry have otherwise fairly normal pregnancies, although they have an increased risk of premature birth and low-birth-weight infants. The 2 major causes of miscarriage are uterine malformations and chromosomal abnormalities. There are several stages of miscarriages. An inevitable abortion is defined by 1st trimester bleeding with an open internal os and no passage of placental or fetal parts.

A 26-year-old athlete presents complaining of scant menses x 4 months. She is a G0P0, menarche was at age 13, and her menses have been mostly regular. She is a non-smoker and non-drinker, and has been trying to achieve pregnancy x 8 months. Physical exam reveals a thin, white female in no distress. Vitals are normal, BMI is 17.5, her pelvic exam is normal, and STD cultures are negative. A pregnancy test is negative in clinic. What is the most likely cause for her amenorrhea? A. Idiopathic B. Hypothalamic C. Hypothyroid D. Polycystic ovarian syndrome E. Androgenic

B. Hypothalamic A young healthy woman with a low BMI, no other signs of virilization, and a history of normal menarche is most likely to have induced amenorrhea, due to suppression of the hypothalamic axis from low weight and fat index.

A 19-year-old sexually active woman presents to your office with questions about cervical cancer screening. She wants to know when she should start getting screened since she's been sexually active for two years. Which of the following is the most appropriate next step in management? A. Initiate screening at age 21 with cytology and human papillomavirus testingYour Answer B. Initiate screening at age 21 with cytology only C. Initiate screening now with cytology and human papillomavirus testing D. Initiate screening now with cytology only

B. Initiate screening at age 21 with cytology only

A 19 year old female presents for her first pelvic examination and is noted to have a palpable, non-tender right ovary. She is sexually active, and uses condoms for birth control and protection from STDs. Her LMP was 20 days ago. She underwent transvaginal ultrasound and was found to have a 3-cm follicular cyst. What does this right ovarian mass represent? A. Retained hemorrhagic products B. Intrafollicular fluids C. Solid intracellular material D. Germ cell layers E. Keratinized squamous epithelium

B. Intrafollicular fluids Follicular cysts occur prior to ovulation and cause expansion of the follicular antrum, serious fluid collection, and subsequent follicular cyst formation. Corpus luteum cysts form after ovulation and may accumulate blood products. Germ cell layers and keratinized squamous epithelium are solid components of teratomas. (Schorge et al., Williams Gynecolog

A 14-year-old woman presents to clinic with some frustration over never having a menstrual period. She is short in stature and has Tanner stage 2 breast development. As you begin a gynecological exam, you realize that you cannot pass a speculum into the vagina. Which of the following is the most likely diagnosis? A. Dysfunctional uterine bleeding B. Primary amenorrhea C. Secondary dysmenorrhea D. Sheehan's syndrome

B. Primary amenorrhea Primary amenorrhea, seen in approximately 2.6% of the population, is defined as absence of menarche by age 16 in a woman with normal growth and secondary sexual development, or age 14 in a woman without normal growth and secondary sexual development. Secondary amenorrhea, in women who have previously menstruated, is defined as absence of menses for more than 3 cycles or 6 months. The most common cause of primary amenorrhea is caused by gonadal dysgenesis due to a chromosome abnormality, while other causes include hypothalamic disease, pituitary disease, abnormal hymen (as in the patient above) or vagina development or uterine agenesis. The patient may have a family history significant for sexual development abnormalities. The most common cause of secondary amenorrhea is pregnancy, followed by abnormalities of the hypothalamic-pituitary-ovary axis, thyroid disease, and ovarian or uterine disorders.

A 30-year-old woman, who had an uncomplicated pregnancy, delivered a healthy newborn a few minutes ago. She just now delivered a placenta with absent cotyledons. Copious amounts of blood begin to flow from the vaginal orifice. Within minutes, the patient becomes lightheaded but maintains consciousness. You attempt manual extraction but the bleeding continues and the blood pressure decreases to 98/68 mmHg. Which of the following is the next best step in management of postpartum hemorrhage? A. Blood type and cross-match B. Suction curettage C. Uterine artery ligation D. Uterine packing with gauze

B. Suction curettage Postpartum hemorrhage can be a serious, and sometimes fatal, situation. One of the causes is retention of placental tissue, which is usually due to abnormal placental implantation or an abnormal separation process. Delivery of an incomplete placenta, for example one that is abnormally shaped or missing some of its normal septations (cotyledons), may prevent normal postpartum uterine contractions. This can lead to improper constriction of the spiral arteries (uterine atony), and ultimately, excessive bleeding. If retained placenta is a highly suspected cause of postpartum hemorrhage, immediate digital extraction should be performed by inserting fingers thru the cervix and into the uterus, then using them to direct and maneuver any remaining intrauterine tissue out through the vagina. If this is unsuccessful, and bleeding continues or the patient decompensates, curettage with a suction device or sharp curette is recommended. Adjuvant interventions include establishing large-bore intravenous access, uterine massage, oxytocin, methylergonovine maleate and alerting the operating room. Obtaining a blood type (A) and alerting the blood bank is wise in managing any patient with hemorrhage. However, in this case, it should not supersede a definitive treatment like curettage.

A 25-year-old woman and her husband have been using condoms and spermicidal foam for the 8 months since the birth of their baby. She plans to wean the baby from the breast sometime between a year and 18 months of age, but would like to begin a "less messy" method of contraception. Prior to her pregnancy she took combination oral contraceptives for several years without any difficulties. Which of the following is an appropriate recommendation for this woman? A. "Breast feeding alone will prevent pregnancy until the baby is weaned." B. "Condoms and spermicide are your only option until you wean the baby." C. "Progestin-only pills are recommended for women who are breast feeding." D. "A vaginal ring will not affect your milk production." E. "You can start back on the same oral contraceptive you took before."

C. "Progestin-only pills are recommended for women who are breast feeding." The progestin-only pill is ideal for breast feeding mothers because this pill does not interfere with lactation the way combination pills do. Breast feeding (A) is reasonably effective in preventing pregnancy only as long as breast milk is the infant's only source of nutrition. Condoms and spermicide (B) are options, but not the only ones for breastfeeding couples. A vaginal ring (D) does decrease the amount of milk production somewhat, but may be an effective option if lactation is well established. Combination oral contraceptives (E) are not recommended for breastfeeding women.

A 22-year-old female presents to her obstetrical appointment at 39 weeks gestation. Her pregnancy to date has been uncomplicated. She is concerned that her infant may be larger than average, as her fundal height measures 41. On physical exam, her fetus is in a cephalic presentation, her cervix is soft and 1-cm dilated, and the fetus is at a -3 station. Her membranes are intact, she is not contracting, and her vitals are normal. She requests to be induced. For decreased risk of complication and optimal fetal outcome, when should she expect to be induced? A. Now B. At 40.5 weeks C. 41.5 weeks D. 42.5 weeks E. Not expected

C. 41.5 weeks In the absence of complication, the recommendation from ACOG is to wait for labor to occur. Large for gestational age is not an indication for induction in the absence of diabetes. Gestations greater than 42 weeks increase risk of fetal stillbirth.

A 29-year-old G2 P1 who is term requests induction. Her pregnancy has been uncomplicated. She has been bothered by significant, poor quality contractions, which have caused her pain and interfered with her sleep. An elective induction is considered safe when the Bishops score is greater than what number? A. 6 B. 7 C. 8 D. 9 E. 10

C. 8 A Bishop score greater than 9 is considered a positive predictor for safe delivery in a term pregnancy.

A 22-year-old woman presents with lower abdominal pain and abnormal vaginal discharge for 4 days. She is sexually active with multiple partners and does not consistently use barrier contraception. She has bilateral adnexal tenderness and yellow discharge on pelvic exam. Her urine pregnancy test is negative. In addition to a 1-time dose of ceftriaxone, what is the appropriate outpatient course of antibiotics for the patient? A. Azithromycin 1 gram PO x 1 B. Ciprofloxacin 500 mg PO BID x 14 days C. Doxycycline 100 mg PO BID x 14 days D. Metronidazole 500 mg PO BID x 14 days

C. Doxycycline 100 mg PO BID x 14 days The patient's presentation is consistent with pelvic inflammatory disease (PID), which represents a spectrum of disorders usually secondary to 1 or more sexually transmitted diseases involving the upper genital tract of women. PID can include any of the following: mucopurulent cervicitis, endometritis, salpingitis, tubo-ovarian abscess, pelvic peritonitis. Patients typically present with complaints of lower abdominal pain, with or without dyspareunia; abnormal bleeding; or abnormal vaginal discharge. On exam, patients usually have lower abdominal tenderness, cervical motion tenderness, and bilateral adnexal tenderness. Outpatient management is appropriate for mild cases of PID and includes ceftriaxone 250 mg IM in a single dose plus doxycycline 100 mg PO BID for 14 days.

A 46 year-old female has just been diagnosed with ovarian cancer. Where would you expect the ovarian neoplasm to most likely arise from? A. Germ cell B. Stromal cell C. Epithelial cell D. A Metastatic tumor E. Endometrial cel

C. Epithelial cell The most common and most lethal of the ovarian neoplasms arise from the ovarian epithelium found both on the surface of the ovary and in subsurface locations. The ovarian epithelium generally is in good health and appears as a simple epithelium. With neoplastic transformation there undergoes metaplastic changes into what is termed Müllerian epithelium. The Müllerian epithelium has a variety of subtypes, each providing a specific phenotype of the tumor and may have a different clinical presentation. Epithelial tumors are the most common ovarian neoplasm and may be: benign (50%); malignant (33%); or borderline malignancy (16%). Tumors may also metastasize (D) to the ovary from the breast, gastric, pancreatic, and colon primary cancers, but is not as common. Germ cell tumors (A) are more similar to testicular tumors in males. Stromal tumors (B) arise from steroid hormone producing cells. Endometrial (E) is usually specific to the uterus.

A 26-year-old female presents to clinic complaining of increasing headaches for one year, irritability, bloating and fluid retention, and abdominal discomfort with loose stools during her menstrual cycle. The symptoms begin a day or two before her menses, and last until the middle of her cycle. She has tried acetaminophen and ibuprofen without improvement. On physical exam she is a well-developed, well-nourished female in no acute distress. Vitals are normal, CV and lungs are normal, pelvis exam is normal, pap smear is normal, and GC and Chlamydia testing are negative. What would be the most appropriate next step? A. Pelvic ultrasound B. FSH, LH levels C. Fluoxetine on cycle day 21-7 D. Paroxetine daily E. Serum HCG

C. Fluoxetine on cycle day 21-7 Hx and Px are key to diagnosing premenstrual syndrome. Laboratory and radiologic procedures are not useful, particularly in light of a normal exam. With failure of NSAIDS, treatment is aimed at reducing symptoms. For mild to moderate symptoms, SSRI therapy prior to and through the menstrual cycle has become a primary therapy.

What is the most frequent cystic structure found in the ovary? A. Corpus luteal cyst B. Dermoid cyst C. Follicular cyst D. Theca lutein cyst

C. Follicular cyst Follicular cysts are the most frequent cystic structures in normal ovaries. They may be found as early as 20 weeks gestation in female fetuses and throughout a woman's reproductive life. Follicular cysts are frequently multiple and may vary from a few millimeters, to as large as 15 cm in diameter. A normal follicle may develop into a physiologic cyst. Follicular cysts are not neoplastic and are believed to be dependent on gonadotropins for growth. These cysts are found most commonly in young, menstruating women. Follicular cysts are translucent, thin-walled, and are filled with a watery, clear to straw-colored fluid. Most follicular cysts are asymptomatic and are discovered during ultrasound imaging of the pelvis or a routine pelvic examination. Ultrasound cannot differentiate a benign from a malignant process.

A 24-year-old G1P1 presents to the office complaining of a red, tender area of her right breast. She is four weeks postpartum and is nursing her infant with good success. She complains of no other symptoms. On physical exam, her vitals are normal. Lungs CTA, CV RRR, left breast is normal, right breast has a 3-cm area that is warm with erythema, and no mass or area of fluctuance is noted. She has a MRSA mastitis. How did she most likely contract the infection? A. Community acquired B.. Hospital acquired C. From her infant D. Self inoculated

C. From her infant Infants usually contract MRSA due to poor hand washing technique from the hospital staff, but it is then spread to the mother via the infant.

A 20-year-old woman was just told by her new sexual partner that she needed to be checked for a sexually transmitted infection because he has developed dysuria and a profuse urethral discharge. She herself has had a subjective fever for the past two days, some nausea but no vomiting, diffuse lower abdominal pain, and a severe backache. On examination, she has a temperature of 100.5˚F, hypoactive bowel sounds, bilateral lower abdominal quadrant tenderness, a profuse mucopurulent cervical discharge and pronounced cervical motion tenderness. Serum pregnancy testing is negative. She is given an injection of ceftriaxone and a prescription for doxycycline for 14 days and an appointment for follow up the next day. Under which of the following conditions should metronidazole be added to her regimen? A. If she does not appear improved by the following morning B. If she develops vomiting or diarrhea C. If she has a probable tubo-ovarian abscess D. If she has an allergic reaction to the doxycycline E. If she has had more than one partner in the past month

C. If she has a probable tubo-ovarian abscess Recommended regimens for treatment of pelvic inflammatory disease include ceftriaxone or another parenteral third-generation cephalosporin and doxycycline or cefoxitin, probenecid, and doxycycline. Metronidazole (or clindamycin) should be added to either regimen if a tubo-ovarian abscess is present. If being treated as an outpatient and she has not improved in 3 days (A), or if she develops vomiting (B), she should be hospitalized rather than given metronidazole. Metronidazole is not a satisfactory substitute for doxycycline (D). The patient's condition rather than the number of sexual partners (E) determines the appropriate therapy.

A 24-year-old gravid 3 para 0 ab 2 presents complaining of vaginal pressure every 2 hours, but no pain. She is at 26 weeks gestation. This pregnancy is uncomplicated to date. On physical exam she is in no acute distress and her vitals are normal. She is placed on a fetal monitor and no contractions are noted; fetal heart tones are 138. A sterile speculum exam reveals her cervix to be dilated to 4 cm. What is the most likely diagnosis? A. Preterm labor B. Group B Streptococcus C. Incompetent cervix D. Braxton Hicks contraction E. Chlamydia trachomatis

C. Incompetent cervix While infection is a significant cause of premature labor, it is not a cause of incompetent cervix. Incompetent cervix is most likely found in the presence of recurrent pregnancy loss and painless dilatation.

At her routine annual pelvic exam, a 39-year-old female presents to the clinic complaining of pelvic pressure and bloating for several months. She is a G3P2 who delivered vaginally. She is a nonsmoker. Her maternal aunt had a history of ovarian cancer. Her pelvic exam reveals an 8-cm ovarian mass in the right adnexal area. What is the most appropriate evaluation of the ovarian mass? A. Pelvic ultrasound B. Pelvic CT scan C. Surgical evaluation D. CA 125 level E. Repeat pelvic exam 1 month

C. Surgical evaluation The patient is high risk, as she is premenopausal, has a family history of cancer, and the mass is large. Therefore, surgical evaluation should be undertaken. CA 125 can be negative in early disease, and pelvic US and CT are not sensitive enough. Repeat examination should be reserved for low risk women with smaller ovarian masses.

A 26-year-old woman presents for her annual exam and is inquiring about birth control. She is current on her immunizations and her last pap smear was 2 years ago. She is in a monogamous relationship with her boyfriend and does not have any immediate plans for pregnancy. She has a history of migraines with aura but does not require any prescription medication. Which of the following birth control methods is the best option for her? A. Behavioral methods such as the withdrawal method and periodic abstinence B. Etonogestrel/ethinyl estradiol vaginal C. Medroxyprogesterone acetate D. Norelgestromin/ethinyl estradiol transdermal

C. Medroxyprogesterone acetate Contraception counseling should be routinely performed in all women of child-bearing age at every annual visit. There are many options such as behavioral, barrier and pharmacological methods. However, pharmacological therapy has the highest rate of pregnancy prevention and should be recommended in all women, unless there are contraindications. This patient has a history of migraines with aura which is considered a contraindication to estrogen use. Other contraindications to estrogen are history of deep vein thrombosis, breast cancer within the past 5 years, cigarette smoking in women more than 35 years of age who smoke more than 15 cigarettes per day, ischemic heart disease, stroke, active liver disease, major surgery with prolonged immobilization and poorly controlled hypertension. In these cases, progesterone only therapy (medroxyprogesterone) should be used in the form of the intrauterine device, the injection or the subcutaneous implantation in the arm.

A 24-year-old G2P2 delivered a viable female infant (8 lb 4 oz) via caesarean section, after a failed 20-hour induction for post date pregnancy. On day 2, she developed a postoperative fever of 101F (38.3C). She had slightly increasing abdominal cramping and pain, no change in loci, is voiding well, and has passed flatulence. Her WBC is 19,000. What is the mostly likely cause for her fever? A. Urinary tract infection B. Ileus C. Metritis D. Atelectasis E. Tubo-ovarian abscess

C. Metritis The patient is passing urine and flatulence well, making choices A and B less likely. Fever greater the 38C is the most important indicator of metritis. Fever is not usually indicative of mild atelectasis. Tubo-ovarian abscess is usually a complication from PID. The patient has many risk factors for metritis including c-sect, prolonged induction, and fever.

A 30-year-old woman misses work and presents to the Emergency Department with severe pelvic pain rated at 6/10. She states it began yesterday with the onset of menstruation. She has regular cycles with normal blood flow amount, but has not had this pain before. She denies spine, urologic and rectal symptoms. Vital signs are normal, and physical examination as well as pelvic ultrasound is unremarkable. Serum beta-hCG is negative. Other than referral to a gynecologist for further evaluation, which of the following is the most appropriate initial treatment? A. Intravenous morphine B. Oral contraceptive pills C. Oral ibuprofen D. Oral pregabalin

C. Oral ibuprofen This patient's working diagnosis is most concordant with primary dysmenorrhea, in which there is significant pain associated with the first few days of menses which alters normal activity or requires pain medication to control. Although secondary causes such as endometriosis and pelvic inflammatory disease are the most common misdiagnosis of primary dysmenorrhea, her stable presentation in this specialized setting, in conjunction with a normal initial diagnostic test, favors symptomatic control and follow-up evaluation. To treat dysmenorrhea, non-steroidal antiinflammatory medications (NSAIDs) and acetaminophen are helpful.

A 30-year-old patient presents to labor and delivery complaining of bright red vaginal bleeding. She has no pain. The fetus is still active. She is 37 weeks pregnant. PMH is significant for in vitro fertilization. What is the most likely diagnosis? A. Placental abruption B. Placenta acreata C. Placenta previa D. Disseminated intravascular coagulopathy E. Active labor

C. Placenta previa The most likely diagnosis is placenta previa, as the bleeding is bright red and painless. Labor and abruption are associated with discomfort and pain.

A 15-year-old girl complains of vaginal discharge over the past two weeks. She reports recently becoming sexually active but uses condoms consistently during intercourse. Which of the following favors a diagnosis of bacterial vaginosis over trichmoniasis? A. Improvement on oral metronidazole B. Multiple punctate hemorrhagic cervical lesions C. Predominance of gram negative rods on gram stain D. Vaginal pH of 5.5

C. Predominance of gram negative rods on gram stain A predominance of gram-negative rods on gram stain is characteristic of bacterial vaginosis, an overgrowth of vaginal anaerobes. Women with bacterial vaginosis present with mild-to-moderate grey, homogenous discharge without pain or irritation of the vulva. A characteristic "fishy" amine odor is noted upon combination with potassium hydroxide. Under microscopy, vaginal secretions characteristically show "clue cells," squamous cells studded with bacteria. Gram stain of discharge reveals gram negative rods, although gram stain is rarely necessary to make a diagnosis.

A 3-year-old girl presents to the ED with her mom and grandmother for evaluation of vaginal spotting. The child has no other complaints and specifically denies sexual abuse or trauma when questioned alone. After watching a news special on sexual abuse, mom is concerned that her daughter may have been sexually assaulted, despite no specific concern. The child is acting appropriately and there are no external lesions or signs of trauma. Internal pelvic exam is difficult due to the patient's age. Which of the following statements is true? A. Obtain a pelvic x-ray prior to attempting a physical exam B. She is the victim of abuse; do no further evaluation without a trained nurse present and contacting authorities C. She may have a vaginal foreign body; consider a nasal speculum to attempt visualization and removal D. Vaginitis is an unusual diagnosis in this age group

C. She may have a vaginal foreign body; consider a nasal speculum to attempt visualization and removal Children often fear parental disapproval of a vaginal foreign body placement. This often leads to a delay in diagnosis until secondary signs (such as vaginal bleeding, foul smelling discharge, or purulent drainage) are noted by parents. The physical exam is often quite difficult owing to patient anxiety, small anatomic size, and parental concerns about sexual or physical abuse. However, a thorough vaginal exam is indicated in patients with vaginal bleeding or discharge. Use of a nasal speculum, procedural sedation, or trained assistant (such as a child life specialist) may facilitate this exam.

A 19-year-old presents to clinic requesting emergency contraception. She is a G1P0Ab1 and a non smoker who has had intercourse and the condom broke. Her LMP was 3 weeks ago. Her PMH is negative. What would be the time frame for maximum efficacy for her to use emergency contraception? A. 12 hours B. 24 hours C. 48 hours D. 72 hours E. 120 hours

D. 72 hours Emergency contraception, both hormonal and IUD, reduces pregnancy rates for 120 hours, but there is a significant decrease in efficacy after 72 hours.

A 40-year-old woman with a history of asthma presents to the ED with symptoms of wheezing and shortness of breath similar to previous exacerbations. Her vital signs are BP 115/70, HR 80, RR, 14, and pulse oximetry is 99% on room air. The patient is offered and agrees to a point-of-care beta-hCG test that returns positive. On exam, you note mild bilateral wheezing with good air movement. Pelvic exam reveals a closed os without adnexal tenderness or masses. Which of the following is the most appropriate next step in management? A. Delay treating her asthma until her pregnancy status is further clarified B. Treat her asthma as indicated, and perform a beta-hCG quantitative level C. Treat her asthma as indicated, if improved, discharge with outpatient obstetrical follow-up D. Treat her asthma as indicated, perform a beta-hCG quantitative level, and obtain a pelvic ultrasound

C. Treat her asthma as indicated, if improved, discharge with outpatient obstetrical follow-up As a good public health practice, many EDs offer routine screening for pregnancy using point-of-care testing. In this patient—with isolated respiratory complaints and lack of findings on physical or pelvic exam that indicate anything other than a normal pregnancy—an incidental positive beta-hCG does not warrant any further attention other than referral for follow-up with an obstetrician. The patient's asthma is the treatment priority. Standard therapy should be administered even in the setting of pregnancy.

A 32-year-old woman 8 weeks pregnant by dates presents to the ED with a 2-cm laceration to her index finger sustained while she was cutting a tomato. On review of systems, she also notes 2 days of vaginal spotting and lower abdominal cramping. Vital signs are within normal limits. Physical exam is consistent with a simple 2-cm laceration. The pelvic exam reveals a closed os and no adnexal tenderness or masses. Which of the following statements best describes the next step in management? A. Delay treating her laceration until her pregnancy status is further clarified B. Treat her laceration as indicated, and perform a beta-hCG quantitative level C. Treat her laceration as indicated, perform a beta-hCG quantitative level, and obtain a pelvic ultrasound D. Treat her laceration as indicated, then discharge with outpatient obstetrical follow-up

C. Treat her laceration as indicated, perform a beta-hCG quantitative level, and obtain a pelvic ultrasound Women aged 21-65 should undergo screening with cytology using the Pap smear every three years. Women aged 30-65 years who want to increase the interval of testing may add HPV testing to the Pap smear and undergo co-testing every five years. Abnormal results require individualized adjustments to the screening schedule.

A postmenopausal patient of yours is diagnosed with atrophic vaginitis and dyspareunia. She denies other menopausal complaints. Vaginal lubricants and moisturizers are not helping to decrease her symptoms. Which of the following would be most appropriate to prescribe next? A. Oral estrogen B. Oral progesterone C. Vaginal estrogen D. Vaginal progesterone

C. Vaginal estrogen The symptoms of vaginal atrophy, or atrophic vaginitis, can be quite bothersome and uncomfortable. They include irritation, burning, bleeding, dryness and dyspareunia, and are common in postmenopausal women. Treatment goals are based on relief of these symptoms. Regular sexual activity can help keep vaginal epithelium healthy. First line therapy includes vaginal lubricants and moisturizers. If patients fail these, consider vaginal estrogen instead of oral estrogen preparations.

A 30-year-old female G2 P2, who delivered via normal spontaneous vaginal delivery, presents complaining of increasing vaginal pressure, low back pain, and stress incontinence. What is the mostly likely cause of her condition? A. Damage to the levator muscles B. Increased intra abdominal pressure C. Widening of the levator gap D. Widening of the AP pelvis diameter E. Endopelvic fascia remodeling and cervical elongation

C. Widening of the levator gap Damage to the levator and increased abdominal pressure are known risk factors, but the widening of the gap with the associated risk factors is what allows the defect to occur. Fascia remodeling can occur instead of a prolapse defect.

A 27-year-old female is 8 weeks postpartum with her first child and has been exclusively nursing since discharge at the hospital. She has a 5-day history of engorgement in her right breast, which is red, tender, and feels warm to the touch. She states she is feverish but has not taken her temperature. She reports no known drug allergies. On physical examination you see the breast as shown below. Which of the following is the most appropriate therapy in the management of this patient? A. Cephalexin 500 mg every 6 hours for 3 days; discontinue breastfeeding B. Cephalexin 500 mg every 6 hours for 3 days; continue breastfeeding C. Discontinue breastfeeding and see if symptoms resolve D. Dicloxacillin 500 mg orally every 6 hours for 10 days; continue breastfeeding E. Dicloxacillin 500 mg orally every 6 hours for 10 days; discontinue breastfeeding

D. Dicloxacillin 500 mg orally every 6 hours for 10 days; continue breastfeeding Staphylococcus aureus is usually the causing agent in puerperal mastitis. Treatment includes antibiotics that are effective against penicillin-resistant staphylococci and nursing of the infant with the affected breast is safe. Alternatively, a cephalosporin can be given orally for 10-14 days. Cephalexin 500 mg every 6 hours (A and B) can be used but should be continued for longer than 3 days. Failure of the patient to respond to antibiotics after three days becomes concerning for an abscess or resistant organism. Further evaluation would be needed including surgical consultation, ultrasound, and hospital admission for IV antibiotics. Dicloxacillin is commonly used in uncomplicated cases without any known allergies. To preserve milk supply, it is encouraged to continue breastfeeding with mastitis; do not recommend discontinuing breastfeeding (A, C, and E).

You have been monitoring a 52-year-old perimenopausal woman's hot flashes. She has not had a hysterectomy. Her symptoms have been so mild that she does not require medication. However, for the past two months, her hot flashes have increased in frequency, duration and intensity. She is now asking for a medication. Which of the following is the most appropriate for medical management of her moderate to severe symptoms? A. Androstenedione alone B. Androstenedione plus estrogen C. Estrogen alone D. Estrogen plus progestin

D. Estrogen plus progestin Most women with mild symptoms do not require medication management. When symptoms become moderate to severe, estrogen therapy is recommended, either oral or transdermal. In women who have not had a hysterectomy, estrogen plus progestin is recommended to prevent endometrial hyperplasia. Androstenedione (A & B) is a precursor hormone in the biosynthesis of estrogen. Its transformation requires an ovary to be receptive to luteinizing hormone, a process which does not occur in menopause. Estrogen alone (C) is the treatment in women who have had a hysterectomy.

An 18-year-old G1P0 presents for evaluation of her amenorrhea. Her LMP was 2 months ago. Her HCG is positive, and she has been having vaginal spotting x 2 weeks. Her ultrasound reveals an intrauterine heterogeneous echogenic mass, without fetus or placenta. What is the most likely diagnosis? A. Threatened abortion B. Missed abortion C. Ectopic pregnancy D. Hydatidiform mole E. Early intrauterine pregnancy

D. Hydatidiform mole The classic signs for a mole are a heterogenous mass without the placenta or fetus, and vaginal spotting is present all of the time. The presence of the mass intrauterine without fetus or placenta rules out ectopic and IUP, and therefore threatened abortion. Missed abortion is not echogenic.

A 30-year-old woman comes in for evaluation of infertility. She and her husband have been having unprotected intercourse for the past year. Her menstrual cramps have become increasingly painful, and she has a severe low backache for several days before and during her menses. She complains that intercourse is painful when her husband "goes deep." Physical examination reveals multiple tender nodules of various sizes in the posterior vaginal fornix. Definitive diagnosis is best accomplished using what methodology? A. Abdominal radiography B. CA-125 measurement C. CT scanning of the abdomen D. Laparoscopy E. Pelvic ultrasonography

D. Laparoscopy This woman has a classic presentation for endometriosis. Definitive or final diagnosis can only be made at laparoscopy or laparotomy, allowing direct visualization of the endometrial implants.

A 22-year-old woman at 36 weeks gestation presents with complaints of feeling a sudden gush of water coming from her vagina. She has had regular obstetrical follow-up and a normally progressing pregnancy. Which of the following is the most appropriate next step in management? A. Administration of corticosteroids B. Immediate digital examination of the cervix C. Initiation of magnesium sulfate infusion D. Microscopic evaluation of vaginal fluid

D. Microscopic evaluation of vaginal fluid Premature rupture of membranes (PROM) is the rupture of the fetal membranes before the onset of labor. In most cases, this occurs near term. When membrane rupture occurs before 37 weeks gestation, it is known as preterm PROM. Preterm PROM occurs in 3% of pregnancies and is the cause of approximately 33% of preterm deliveries. The presence of amniotic fluid can be confirmed with nitrazine testing (a pH of 7.1-7.3 as opposed to a normal pH of 3.5-6.0), the presence of ferning on microscopic evaluation, or smear combustion (amniotic fluid turns white and crystalizes when flamed).

A 18-year-old G1P0 woman at 16-weeks presents with vaginal bleeding. She had no prenatal care. Vital signs are unremarkable and physical examination only reveals a small amount of blood in the vaginal vault. A transvaginal ultrasound is performed as seen above (snowstorm). Which of the following managements is most likely indicated? A. Administer methotrexate B. CT scan of the abdomen and pelvis C. Intravenous antibiotics D. Obstetrics consultation

D. Obstetrics consultation This patient presents with vaginal bleeding and an ultrasound consistent with a hydatidiform mole or molar pregnancy requiring obstetrics consultation. Molar pregnancy is a spectrum of diseases characterized by abnormal chorionic villi proliferation. A complete hydatidiform mole refers to the situation in which there is no fetal tissue. In an incomplete mole, there is some fetal tissue along with trophoblastic hyperplasia. Patients with molar pregnancy may present with nausea, vomiting, abdominal pain, and vaginal bleeding. Without ultrasound, it is difficult to differentiate these patients from a threatened miscarriage or ectopic pregnancy. Often, the uterine size is larger than the expected for dates in molar pregnancy and the beta-hCG is higher than expected for dates. Diagnosis is based on characteristic findings on ultrasound. Hydropic vessels within the uterus cause a "snowstorm" appearance. Because of the potential for complications and the non-viability of the pregnancy, dilation and curettage is recommended.

A 30-year-old woman presents with abnormal menstrual bleeding. Her history and physical are unrevealing. Other than laboratory testing, which of the following is the next best step in evaluating this patient? A. Dilatation and curettage B. Endometrial biopsy C. Hysteroscopy D. Pelvic ultrasonography

D. Pelvic ultrasonography Determining the source of abnormal uterine bleeding is a difficult task when caring for gynecologic patients. This evaluation begins with a detailed history and physical followed by laboratory tests and pelvic ultrasound. Common initial laboratory tests include a complete blood count, serum prolactin level, serum ß-hCG and serum thyroid stimulating hormone level. Pelvic ultrasonography may reveal uterine lining abnormalities, including irregularities, polyps, fibroids and masses. The results of the above will guide the clinician down a non-gynecologic versus gynecologic pathway. If a gynecologic source is suggested, endometrial biopsy (B), which can be accomplished via many different curette procedures, is then recommended. If the source of bleeding is still not found, hysteroscopy (C), which allows direct visualization of the uterine cavity, is then recommended. If the diagnosis is still unclear, the patient may be sent for dilatation and curettage (A). This procedure's sedation allows for muscular relaxation which may increase diagnostic yield.

A 15-year-old female presents with generalized abdominopelvic pain which occurs every month after her regular, nonpainful menses. The pain, which is associated with headaches, bloating and depressed mood, begins 18 days after the last day of menstruation. She also complains of cyclic ankle swelling but denies a history of increased salt intake. Which of the following is the most likely diagnosis? A. Amenorrhea B. Dysfunctional uterine bleeding C. Dysmenorrhea D. Premenstrual syndrome

D. Premenstrual syndrome This patient displays some of the common symptoms of premenstrual syndrome, a poorly understood condition of physical, mood and behavioral changes which occur during the second half of the typical 28-day menstrual cycle. On average, menstrual flow lasts 4-5 days, beginning on day 1 of the menstrual cycle (the beginning of the follicular phase), and ending on day 4 or 5. Around day 14, ovulation occurs, and the second half of the cycle, the luteal phase, begins. If the oocyte is not fertilized, the luteal phase, on average, ends around day 28. Subsequently, a new cycle commences with the onset of another menses. The symptoms of premenstrual syndrome occur in this second half period (day 14-28), most commonly occurring on day 23-27, which is typically 18-19 days after the last day of the most recent menstruation.

A 32-year-old woman presents to your office for a physical exam including a Papanicolaou test (Pap smear). Lab results reveal negative cytology and positive human papillomavirus (HPV). Which of the following is the most appropriate next step in management? A. Counsel patient on safe sex practices B. Order colposcopy C. Repeat Pap smear and human papillomavirus testing in five years D. Repeat Pap smear and human papillomavirus testing in one year

D. Repeat Pap smear and human papillomavirus testing in one year Cervical cancer screening is done through the use of the Papanicolaou (Pap) test for cytology and testing for high-risk strains of the human papillomavirus (HPV). Initial screening begins at age 21, regardless of sexual activity. Type and frequency of testing then depend on the patient's age. Women under 30 years should be screened with the Pap smear alone every three years. Women 30 and older should be screened with Pap smear and HPV co-testing. If the Pap smear test is negative and HPV test is positive, providers may either repeat Pap smear and HPV co-testing in one year or order HPV DNA typing to detect HPV subtypes 16 or 18. These subtypes are associated with the highest risk of cervical cancer and are important to identify early, allowing for prompt intervention.

A 24-year-old woman presents for initial evaluation of cyclical symptoms of irritability, painful bloating and depression. These symptoms occur regularly 4-5 days prior to the beginning of menstruation. During these few days, this patient typically has to miss work due to the "awful pain and mental clouding." Which of the following treatment options do you recommend as first-line therapy? A. Anticholinergics B. Hysterectomy C. Oral contraceptive pills D. Selective serotonin reuptake inhibitors

D. Selective serotonin reuptake inhibitors Treatment of mild premenstrual syndrome symptoms, those considered to not alter daily socioeconomic function, includes stress reduction techniques and regular exercise. This patient reports historical points which reveal moderate to severe symptomatology. In women whose daily function is altered due to the cyclical, luteal phase symptoms of premenstrual syndrome, selective serotonin reuptake inhibitors (SSRIs) are recommend as first-line therapy. Options include sertraline or fluoxetine, administered on a daily basis or only during the second half (luteal phase) of the patient's menstrual cycle.

A woman presents for a routine post-partum checkup four weeks after delivery of her child. She is currently breast feeding without difficulty. She tells you that she has been feeling anxious and very warm, despite the change of season into winter. She is happy as a mother and has not had problems caring for her baby. What is the next best step in the investigation of her symptoms? A. Radioactive iodine uptake test B. Serum total T4 level C. Thyroid fine needle aspiration D. Serum TSH level E. MRI of the anterior pituitary

D. Serum TSH level The correct choice is D, serum TSH level. This patient is presenting with post-partum thyroiditis. Thyroid dysfunction occurs in 2 to 5% of women after giving birth. It can recur with subsequent pregnancies and develop into long-term thyroid disease as well. Typically, women with this disorder first develop signs and symptoms of hyperthyroidism, which then later changes to hypothyroidism. Most symptoms resolve spontaneously within a few months. In the hyperthyroid stage of this disorder, blood tests will reveal a suppressed TSH level with elevated serum thyroid hormone levels, as is common in all forms of primary hyperthyroidism. Choice A, a radioactive iodine uptake test, would reveal little or no uptake, but this test should not be ordered in a woman who is breastfeeding. For choice B, a serum total T 4 level can be elevated, but is not as sensitive or specific as TSH as a screening test. Any protein status changes in the woman can cause an elevated total T 4 without any thyroid dysfunction. Choice C, a thyroid fine needle aspiration, can be performed, but is not likely necessary and would not be the best next step for this patient. Choice E, an MRI of the anterior pituitary, would only be suggested if there is a suspicion of a pituitary tumor. This is a rare cause of hyperthyroidism and would not be the best next step.

A 25-year-old G2P1 presents to your office at 32 weeks gestation with a complaint of severe itching, particularly on the palms of her hands and the soles of her feet. Lab results reveal elevated bile acids. Regarding this disease, which of the following statements is most correct? A. Aminotransferases are low B. Disease recurrence is rare in subsequent pregnancies C. The treatment of choice is cholestyramine D. There is an increased risk for fetal demise

D. There is an increased risk for fetal demise This woman has classic symptoms of intrahepatic cholestasis of pregnancy. It is characterized by pruritus which is often concentrated in the palms of the hands and soles of the feet. Serum bile acids are almost always elevated and there is a significant increase in intrauterine fetal demise. The pathophysiology of fetal death is poorly understood, but may be related to fetal dysrhythmias or placental vasospasms from high bile acid levels. There is no ideal antepartum testing strategy, but regular non stress tests and biophysical profiles looking for signs of fetal compromise are often performed. A widely accepted management approach is induction of labor between 36 and 37 weeks gestation as most fetal deaths occur after 37 weeks. The treatment of choice is ursodiol which helps increase hepatic bile flow and decrease bile acid levels.

A 31-year-old woman at 35-weeks gestation presents with brief painless, bright red vaginal bleeding. In addition to fetal monitoring, which of the following is the most important initial management? A. Administration of betamethasone to hasten fetal lung maturity B. Sterile digital cervical exam C. Sterile speculum examination D. Transvaginal ultrasound

D. Transvaginal ultrasound The clinical scenario of painless, bright red vaginal bleeding is most suggestive of placenta previa. In placenta previa, the placenta abnormally overlies the cervical os. Most cases of placenta previa diagnosed on 20-week ultrasound resolve in the months prior to delivery as the lower uterine segment elongates and the placenta no longer overlies the cervical os. However, in up to 20% of cases the placenta remains positioned over the cervix, which can cause significant and life-threatening hemorrhage at the time of delivery. Transvaginal ultrasound is safe and more accurate than transabdominal ultrasound for diagnosis of placenta previa​.

A 23-year-old woman presents with pain in in the suprapubic area and right lower quadrant of her abdomen that began 4 hours ago. She denies vagina discharge and vaginal bleeding. Her vital signs are BP 115/65, HR 60, RR 12, and T98.1°F. Her last menses was one week ago. Which of the following diagnostic tests should be obtained first? A. Complete blood count B. Gonorrhea and chlamydia DNA amplification C. Urinalysis D. Urine beta-hCG

D. Urine beta-hCG All women of childbearing age presenting with pelvic complaints require a pregnancy test. Even though this patient reports that she had her menses one week prior to her presentation, ectopic pregnancy and even a normal pregnancy can be associated with vaginal bleeding that can be mistaken for normal menses. Since ectopic pregnancy is a life-threatening condition, it is important to rule it out early in this patient's workup. A urine beta-hCG is a rapid test that has a very high sensitivity that can rapidly determine if a woman is pregnant. A negative urine beta-hCG effectively rules out ectopic pregnancy.

Which of the following is the most appropriate treatment for stage IV uterine prolapse in a 50-year-old woman with no medical problems and no previous surgical history? A. Abdominal hysterectomy B. Anterior colporrhaphy C. Posterior colporrhaphy D. Vaginal hysterectomy

D. Vaginal hysterectomy Vaginal hysterectomy is associated with better outcomes and fewer complications than laparoscopic or abdominal hysterectomy.

A 26-year-old obese woman with a 2-year history of increased acne, abnormal hair growth, and menstrual abnormalities presents to her obstetrician for an infertility workup. A pelvic ultrasound reveals enlarged cystic ovaries. She desires to become pregnant. Which of the following is the first line treatment? A. Clomiphene citrate B. Gonadotropin therapy C. Spironolactone D. Weight reduction

D. Weight reduction Polycystic ovarian syndrome (PCOS) is characterized by androgen excess, insulin resistance, and gonadotropin abnormalities. PCOS is the most frequent cause of anovulatory infertility. The diagnostic criteria for PCOS are chronic anovulation and hyperandrogenism (established by hormone measurements or clinical findings such as acne or hirsutism) in women whom secondary causes have been excluded. Polycystic appearing ovaries on ultrasound examination are a nonspecific finding in PCOS. Important factors in the physical examination include the following: the presence of acne, balding, or clitoromegaly; the distribution of body hair; enlargement of the ovaries (based on a pelvic examination); and signs of insulin resistance. Weight reduction is the first line treatment for polycystic ovarian syndrome for women who desire to conceive. As little as 5% total weight reduction has been shown to improve the metabolic and reproductive abnormalities in PCOS.

A concerned mother brings her 13-year-old daughter in for evaluation. All the child's female friends have begun to menstruate, and this child has "barely begun to develop." On examination, the teenager appears well. She is 5'4" tall and weighs 114 pounds. Her breasts demonstrate buds with a small amount of elevation of the breast itself, and fine hair that is just beginning to curl on the labia majora. What is the most appropriate education in this situation? A. She will begin to menstruate in the next month or two. B. Teens who are underweight often develop more slowly than others. C. The other girls must have undergone precocious puberty. D. Your daughter is developing at the time that is right for her. E. We should perform some genetic testing on your daughter.

D. Your daughter is developing at the time that is right for her. Normal puberty occurs within a wide range of ages. Girls are not considered "delayed" until they have shown no sexual development by age 14 or have not menstruated by age 17. This teen is approaching Tanner Stage 3 of development and should menstruate when she has attained Stage 4 (A) in another six months to a year. Very thin or highly athletic girls may not reach puberty as quickly or may cease to menstruate, but this girl's weight is within normal limits for her height. (B) The age of onset of puberty in the developed countries is between 8 and 13 (C). Since this girl is progressing normally and appears well, no genetic testing (E) is indicated.

A progestin-only contraceptive, or "minipill," would be most appropriate for which of the following patients? A. a 25-year-old woman in excellent overall health B. a 28-year-old woman with a history of epilepsy C. a 32-year-old woman with a history of pelvic inflammatory disease D. a 37-year-old woman who smokes 2 packs per day and has a history of hypertension E. a 38-year-old woman with a history of asthma and bronchitis

D. a 37-year-old woman who smokes 2 packs per day and has a history of hypertension In the majority of cases, a combined hormonal contraceptive (ie, one that contains both an estrogen and progestin) is the preferred method of oral contraception because of its efficacy when used perfectly (>99%). However, for women older than 35 years of age who are smokers or are obese, or who have a history of hypertension or vascular disease, progesterone-only contraceptives are recommended. Ethinyl estradiol (EE), the most common estrogen found in combined hormonal contraceptives, has been associated with an increased risk of myocardial infarction in women older than 35 years of age who are smokers. Additionally, EE has also been shown to cause increases in blood pressure in both normotensive and mildly hypertensive women. Progestin-only contraceptives, however, tend to be less effective than the combined hormonal contraceptives.

You are providing care for a 21-year-old G1P1 who delivered a 3990 gram infant by normal spontaneous vaginal delivery. After delivery, she continues to bleed vaginally, more than is expected for routine delivery. What is the most likely cause of her hemorrhage? A. cervical laceration B. lateral vaginal wall laceration C. retained placenta D. uterine atony E. placenta acreata

D. uterine atony Uterine atony continues to be the most common cause of postpartum hemorrhage, even though all of the answer choices may cause it.

A G4P4 woman delivers a viable infant at 38 weeks gestation by normal spontaneous vaginal delivery. The infant has apgars of 7 and 8 (at 1 and 5 minutes respectively). What is the most crucial time for maternal and fetal physiologic changes to occur? A. 30 minutes prior to birth B. 60 minutes prior to birth C. Birth D. 30 minutes after birth E. 60 minutes after birth

E. 60 minutes after birth The hour after birth is the most critical time for physiologic changes to occur, including maternal fluid shifts, hemorrhage, retained placenta, and fetal lung cardiovascular transitions.

During her active phase of labor, a provider has placed an external fetal monitor on a patient. The fetal heart rate (FHR) is noted to have a base line rate of 109, there are no late or variable decelerations, and the baseline variability is 0 beats per minute. What is this monitor tracing indicative of? A. A normal FHR pattern B. An indeterminate FHR pattern C. An unreadable FHR pattern D. Poor contact with monitor E. A distress FHR pattern

E. A distress FHR pattern According to the Three-Tier Fetal Heart Interpretation System, recommended by the 2008 NICHD workshop on electronic fetal monitoring, the definition of fetal distress includes absence of baseline variability, and either bradycardia (FHR<110) or recurrent variable or late decelerations.

A 30-year-old woman presents to the office complaining of an inability to achieve pregnancy after over a year of trying. What is the couple's least likely cause for infertility? A. Azoospermia B. Ovarian failure C. Endometrial disease D. Tubal disease E. Pituitary disease

E. Pituitary disease In the vast majority of women who are infertile or subfertile, the issue is due to either ovarian dysfunction or structural abnormalities, with hormonal abnormalities accounting for only 10%.

A 17-year-old female presents to the emergency department complaining of watery vaginal discharge for 6 hours. She is found to be at 35 weeks gestation. An external fetal monitor reveals fetal heart tones in the 130s, good variability, and no contractions. What is the most likely diagnosis? A. Preterm labor B. Preterm rupture of membranes C. Premature rupture of membranes D. Preterm labor and premature rupture of membranes E. Preterm and premature rupture of membranes

E. Preterm and premature rupture of membranes Preterm rupture of membranes is defined as rupture before 37 weeks gestation; premature is defined as before the onset of labor. The absence of pain or contractions decreases the likelihood of labor.

An 18-year-old woman comes in for evaluation of a bad smelling vaginal discharge. She says that it is so heavy that she must wear a pad to absorb it and that it is "yellowy green and bubbly." She also has a great deal of itching of vulva. Evaluation confirms the presence of a discharge and a vaginal pH of 5.5. Her vagina and cervix are diffusely erythematous with scattered petechiae. What is the most likely diagnosis? A. Atrophic vaginitis B. Bacterial vaginosis C. Candidiasis D. Gonorrhea E. Trichomoniasis

E. Trichomoniasis Trichomoniasis vaginitis is the most common STD in the United States that is not caused by a virus. It causes a profuse and frothy discharge that is greenish and can be foul smelling. Pruritus may occur. The vaginal pH is usually more than 5.0 and the vagina and cervix are red and may demonstrate small petechiae. Atrophic vaginitis (A) occurs primarily in postmenopausal women and is characterized by a thin watery or serosanguinous discharge, while bacterial vaginosis (B) has a greyish homogeneous discharge with a characteristic "fishy" odor released when potassium hydroxide is mixed with vaginal secretions. The discharge of candidiasis (C) is typically white and clumpy and is accompanied by intense itching in the vulva. More than three quarters of women who have gonorrhea (D) have no symptoms, but they may have a mucopurulent discharge.


Conjuntos de estudio relacionados

HSC4713: Planning and Evaluation, HSC 4713 Module 10: Program Resources and Management, HSC 4713 Module 12: Implementation, HSC 4713 - Module 14 Evaluation Approaches and Design, Module 13 - Purposes and Logistics of Evaluation; Evaluation Approaches...

View Set

MANAGEMENT and ORGANIZATIONAL BEHAVIOR

View Set

Chapter 7 Psychology Terms (Thought, Language and Intelligence)

View Set